LSAT and Law School Admissions Forum

Get expert LSAT preparation and law school admissions advice from PowerScore Test Preparation.

User avatar
 Dave Killoran
PowerScore Staff
  • PowerScore Staff
  • Posts: 5853
  • Joined: Mar 25, 2011
|
#74682
Complete Question Explanation
(The complete setup for this game can be found here: lsat/viewtopic.php?f=391&p=94117)

The correct answer choice is (C)

If K is scheduled for Friday, then from the fourth rule V must be scheduled for Thursday. Consequently, only H and I are available to be scheduled for Monday. However, H cannot be scheduled for Monday because from the second rule N would have to be scheduled for Friday, and that is impossible in this question. Thus, only I can be scheduled for Monday:

G2-Q10-d1.png

Accordingly, answer choice (C) is correct.
You do not have the required permissions to view the files attached to this post.
 melissayoung99
  • Posts: 1
  • Joined: Apr 05, 2020
|
#74669
Hello,
I just wanted an explanation as to why the answer cannot be A or E. I marked off B and chose answer E.

Since K must be Friday, I deduced V must be Thursday. This leaves M,Tu,W open. I knew N cannot be Mon (Rule 1) and H cannot be Mon (Rule 2) since if H is assigned M, then N must be F but F is already occupied by K.

So this is the order I get:
I H N V K

Answer choices:
A) H for Tues
C) I for Mon
E) N for Wed

Which is technically all correct...? Please help! Thank you so much!
 Jeremy Press
PowerScore Staff
  • PowerScore Staff
  • Posts: 1000
  • Joined: Jun 12, 2017
|
#74696
Hi Melissa,

Check out the complete question explanation I posted above, and let us know if this resolves your questions!

Thanks!
 bella243
  • Posts: 65
  • Joined: Apr 29, 2020
|
#76347
I solve this somewhat differently, can you please confirm that this is correct? Because K5, No cannot be 5 so H cannot be one according to conditional rules. This means that I must occupy #1 since #1 can be only H or I.
 Adam Tyson
PowerScore Staff
  • PowerScore Staff
  • Posts: 5153
  • Joined: Apr 14, 2011
|
#76468
Exactly right, bella243, and that is what our diagram is meant to illustrate. There are two possible solutions, in both of which I must be on Monday. Well done!

Get the most out of your LSAT Prep Plus subscription.

Analyze and track your performance with our Testing and Analytics Package.